LSAT and Law School Admissions Forum

Get expert LSAT preparation and law school admissions advice from PowerScore Test Preparation.

User avatar
 Dave Killoran
PowerScore Staff
  • PowerScore Staff
  • Posts: 5853
  • Joined: Mar 25, 2011
|
#94176
Complete Question Explanation
(The complete setup for this game can be found here: viewtopic.php?f=385&p=94172)

The correct answer choice is (B)

As determined in the discussion of the numerical distributions, there can be exactly two managers. Thus, answer choice (B) is correct.
 Paul Marsh
PowerScore Staff
  • PowerScore Staff
  • Posts: 290
  • Joined: Oct 15, 2019
|
#78028
This is a Global Could be True question. Again, a quick glance at our answer choices reveals that these answer choices are mostly discussing the possible numeric distributions of our game. So we refresh ourselves on the 1-2-2 and 1-1-3 possible distributions and scan for the answer choice that comports with those two possibilities. (B) fits the bill perfectly, we know that's possible in our 1-2-2 distribution.

Get the most out of your LSAT Prep Plus subscription.

Analyze and track your performance with our Testing and Analytics Package.